What is Conditions: Definition and 1000 Discussions

In mathematical optimization, the Karush–Kuhn–Tucker (KKT) conditions, also known as the Kuhn–Tucker conditions, are first derivative tests (sometimes called first-order necessary conditions) for a solution in nonlinear programming to be optimal, provided that some regularity conditions are satisfied.
Allowing inequality constraints, the KKT approach to nonlinear programming generalizes the method of Lagrange multipliers, which allows only equality constraints. Similar to the Lagrange approach, the constrained maximization (minimization) problem is rewritten as a Lagrange function whose optimal point is a saddle point, i.e. a global maximum (minimum) over the domain of the choice variables and a global minimum (maximum) over the multipliers, which is why the Karush–Kuhn–Tucker theorem is sometimes referred to as the saddle-point theorem.The KKT conditions were originally named after Harold W. Kuhn and Albert W. Tucker, who first published the conditions in 1951. Later scholars discovered that the necessary conditions for this problem had been stated by William Karush in his master's thesis in 1939.

View More On Wikipedia.org
  1. A

    Numerical boundary conditions for wide approximation finite difference

    Hi, I have to use a wide 5 point stencil to solve a problem to fourth order accuracy. In particular, the one I'm using is: u'' = -f(x + 2h) + 16f(x + h) - 30f(x) + 16f(x - h) - f(x - 2h) / 12h2 or when discretized u'' = -Uj-2 + 16Uj-1 -30Uj + 16Uj+1 -Uj+2 / 12h2 In addition to...
  2. F

    How do you build a wave packet and inital conditions

    Hi, I'm trying to get my head around Schrödinger's equation and quantum wave theory. I'll try to shortly state how I understand it so you may see where I'm wrong and better answer the question. In classical mechanics if you solve a linear differential equation, the sum of the solutions is also...
  3. J

    How is the initial conditions for PDE?

    Given a PDE of order 1 and another of order 2, you could show me what is, or which are, all possible initial conditions? For an ODE of order 2, for example, the initial condition is simple, is (t₀, y₀, y'₀). However, for a PDE, I think that there is various way to specify the initial condition...
  4. S

    How to model transient conditions?

    Hello, I am graduate student in NE within the US. I am working on a grant that examines a small (10-100 microns) ceramic film region on the outside of the cladding region. I have assessed the neutronic impact, as well as a thermal impact (heat transfer coefficients, etc.) of the additional...
  5. T

    MHB Differential Approximation with Boundary Conditions

    Hello! I have a nifty set of problems (or rather one problem, gradually building itself to be a great problem) that I like to collectively call "The final problem" as it is the last thing I need before I can take the exam in Numerical Methods.Information There is given a Laplace equation...
  6. kmm

    Electrostatic Boundary Conditions

    In Griffith's section about electrostatic boundary conditions, he says that given a surface with charge density \sigma , and take a wafer-thin Gaussian pillbox extending over the top and bottom of the surface, Gauss's law states that: \oint_{S} \mathbf{E} \cdot d \mathbf{a} =...
  7. S

    Boudary conditions, rough interface (Electrodynamics)

    Hello. I'd like to know, does anybody know any information about finding boudary conditions for E, H vectors in case of rough interface, if phi(x) function is set. Maybe such kind of case ( 2D or 3D case) has been considered somewhere. Thank you in anvance.
  8. G

    Damping conditions for multi degrees of freedom

    I know that d^2<4mk for underdamped, d^2>4mk for overdamped and d^2=4mk for critically damped. This is true if there is only 1 mass and spring and damper. How to use these equations if I have 2 mass, 3 spring and 3 dampers. That is d,m,k are in 2x2 matrices. Please some one help me with this.
  9. H

    Mathematica Mathematica NDSolve conditions help

    Hi, I'm having a problem with NDSolve in mathematica where it is interpreting my initial conditions as "True" or "False". Here is the code: soln = NDSolve[{eqn1[[1]], eqn1[[2]], x[0] == y[0] == 0, x'[0] == 1, y'[0] == 0}, {x, y}, t]; where eqn1 is determined by the following code...
  10. M

    Boundary Conditions on a Penning Trap

    Homework Statement Consider a charged particle, of mass m and charge q, confined in a device called a Penning Trap. In this device, there is a quadrupole electric field described in cartesian coordinates by the potential Phi[x,y,z] = U0 (2z^2 - x^2 - y^2) / (r0^2 + 2z0^2) Where U0 is...
  11. O

    Satisfying Coulomb Gauge: What are the Conditions for Vector Potential?

    Homework Statement We study the free electromagnetic field in a charge and current free cubic box with with edge length L and volume V. The vector potential in such a system is given via Fourier series: Homework Equations \vec{A}(\vec{r}, t) = \sum\limits_{k} \vec{A}_k(t) e^{i...
  12. J

    How do you integrate and differentiate a function with steps and impulses?

    Given x = \left\{\begin{matrix} y \;\;\; case\;A\\ z \;\;\; case\;B\\ \end{matrix}\right. So, operate x means to operate the 2 cases of right side? For example: \int x\;dx = \left\{\begin{matrix} \int y\;dx \;\;\; case\;A\\ \int z\;dx \;\;\; case\;B\\ \end{matrix}\right. Correct?
  13. C

    Gravitational wave solution boundary conditions

    In linearized gravity we can one sets $$(1) \ \ g_{\mu \nu} = \eta_{\mu \nu} + h_{\mu \nu}$$ where h is taken to be a small perturbation about the flat space metric. One common decomposition of h is to write the spatial part as $$ h_{i j} = 2 s_{ij} - 2\psi \delta_{ij} \ h_{0i} \equiv...
  14. A

    Appropriate Boundary Conditions for the Biharmonic Equation

    Usually, when considering the biharmonic equation (given by Δ^2u=f, we look for weak solutions in H^2_0(U), which should obviously have Neumann boundary conditions (u=0 and \bigtriangledown u\cdot\nu =0 where \nu is normal to U). Now consider that we are looking for solutions u\in...
  15. Q

    Boundary conditions of beams

    Homework Statement Just need some quick confirmation. For a beam which has a load applied to it, will its free end always have a shear force, bending moment and curvature of zero?
  16. A

    Determining two sets of boundary conditions for a double integral prob

    Homework Statement Determining two sets of boundary conditions for a double integral problem in the polar coordinate system. Is the below correct? Homework Equations The Attempt at a Solution There are two sets of boundary conditions that you can use to solve this problem in the polar...
  17. G

    Questions on conditions for calculating flux integrals?

    Conditions for calculating flux integrals? [Figured it out] If one uses Stokes' theorem and if two oriented surfaces S1 and S2 share a boundary ∂s then the flux integral of curl(F) across S1 equals the flux integral of curl(F) across S2. However, in general it won't be true that flux integral G...
  18. K

    Solving a Differential Equation with Boundary Conditions

    What is the answer of this differential equation. ((d^2) r)/((ds)^2) +(m/(r^2)) -(nr/3)=0 the boundary conditions (i) r=a when s=0 and (ii) dr/ds =0 when r=b. m and n are constants.
  19. C

    Conditions for a E field to be possible

    If I want to verify an electric field E is possible in free space, do I just need to check ∇xE =0?
  20. L

    Simple initial conditions problem

    Homework Statement http://imgur.com/Mwin7dB http://imgur.com/Mwin7dB Homework Equations The Attempt at a Solution This is a fairly simple problem. My issue is that I can't identify the second initial condition. The first one is simple. At t=0, the voltage on the capacitor is...
  21. I

    Solving 2nd Order Differential Equation with Initial Conditions

    Mod note: Reinstated problem after poster deleted it. Homework Statement Just wondering if I did this correctly: ##y''+4y'+4y=e^{x}## and initial conditions ##y(0)=0; y'(0)=1## Homework Equations The Attempt at a Solution So I found the characteristic equation to be...
  22. S

    Are Fierz and Pauli Conditions Related to Massive Gravity and Higher Spin?

    HI I study some thing about fierz and paulli conditions In massive gravity and Higher spin particle, what's difference between them? the conditions in massive gravity and higher spin are similar. are they related to them? thanks
  23. O

    Boundary conditions of 2 conductors

    Homework Statement Ignore the text in German. You just need to see the picture. 2 conductors both with potential 0 are given. \alpha is the angle between the conductors. (r, \varphi) are polar coordinates pointing to a point in the plane. Homework Equations What we need to do is...
  24. F

    Physical conditions of nebulae/HII regions

    Hello to all! I'm trying to develop a routine (in ESO-MIDAS) similar to NEAT (Nebular Empirical Analysis Tool) or NEBULAR package from IRAF that, by inputting certain emission line ratios i could get the physical conditions of the nebula (like Te, Ne, ionic and elemental abundances). The thing...
  25. Lebombo

    Alternating Series Test Conditions

    Homework Statement This is what I understand about Alternating Series right now: If I have an alternate series, I can apply the alternative series test. \sum(-1)^{n}a_{n} Condition 1: Nth term test on a_{n} Condition 2: 0 < a_{n+1} ≤ a_{n}If condition 1 is positive or ∞, convergence is...
  26. H

    PDE, heat equation with mixed boundary conditions

    Homework Statement solve the heat equation over the interval [0,1] with the following initial data and mixed boundary conditions.Homework Equations \partial _{t}u=2\partial _{x}^{2}u u(0,t)=0, \frac{\partial u}{\partial x}(1,t)=0 with B.C u(x,0)=f(x) where f is piecewise with values: 0...
  27. Jalo

    Quantum mechanics: Find the wave function given the conditions

    Homework Statement Consider a 1-dimensional linear harmonic oscilator. Any measurement of it's energy can either return the value of ħw/2 or 3ħw/2, with equal probability. The mean value of the momentum <P> at the instant t=0 is <P> = (mħw/2)1/2 Find the wave function ψ(x,0) for this...
  28. A

    Redundancy of Lie Group Conditions

    I want to show that if G is a smooth manifold and the multiplication map m:G×G\rightarrow G defined by m(g,h)=gh is smooth, then G is a Lie group. All there is to show is that the inverse map i(g)=g^{-1} is also a smooth map. We can consider a map F:G×G\rightarrow G×G where F(g,h)=(g,gh) and...
  29. J

    MHB Making up an Equation That Satisfies Given Conditions

    Make up an equation that satisfies the given conditions: must have x-intercepts at (-1, 0) and (2, 0). It must also have a y-intercept at (0, 4). Is it correct to say that the equation with x-intercepts would look like this: y = (x - 2) (x + 1)How do you factor the y-intercept?
  30. O

    Damped oscillator given odd initial conditions

    Homework Statement (A) The damped oscillator is described by the equation mx''+bx'+kx=0. What is the condition for critical damping expressed in terms of m,b,k. Assume this is satisfied. (B) For t<0 the mass is at rest (x=0). This mass is set in motion at t=0 by a sharp impulsive force so...
  31. Superposed_Cat

    What are the boundary conditions of the universe?

    What are the boundary conditions of the universe?
  32. E

    Conditions for Index Size in A_5 and S_5 Centralizers

    For any element x \in A_5, we have that [A_5:C_{A_5}(x)]=\begin{cases} [S_5:C_{S_5}(x)], & \text{condition 1} \\ \frac{1}{2}[S_5:C_{S_5}(x)], & \text{condition 2} \end{cases} Basically I want to know what the conditions are. Note that C is the centralizer.
  33. C

    Finding potential given some conditions

    Homework Statement Denote the ground state and the first excited state of a 1D quantum system by ##\psi_{0}## and ##\psi_{1}##. If it is given that $$\psi_{1}(x) = x \psi_{0}(x)\,\,\,\text{and}\,\,V(0)=0$$ find the potential V(x). Homework Equations TISE The Attempt at a Solution If I sub...
  34. Mandelbroth

    General Conditions for Stokes' Theorem

    What is the least restrictive set of conditions needed to utilize the formula ##\int\limits_{\Omega}\mathrm{d}\alpha=\int\limits_{\partial\Omega} \alpha##?
  35. T

    Archived How Does Adding Heat Affect Pressure in a Sealed Steam Container?

    Homework Statement Given: A sealed, rigid container (cannot change shape or size) containing steam at the shown conditions has heat added until it reaches the final shown state. Determine: a) heat added, b) work performed, c) change in internal energy of the system, and d) final system...
  36. anemone

    MHB Finding $f(67500)$ with Given Conditions on $f(x)$

    Given that the function $f(x)$ is defined on the set of natural numbers, taking values from the natural numbers, and that it satisfies the following conditions: (a) $f(xy)=f(x)+f(y)-1$ for any $x,y \in N$. (b) the equality $f(x)=1$ is true for finitely many numbers. (c) $f(90) = 5$...
  37. W

    Ways an organism can survive in extreme conditions.

    Lets ay there is a 0.5 Earth mass and 0.8 Earth radii planet orbiting a Sun like star at the same distance Earth is orbiting its Sun. Its atmospheric pressure would be 0.03 bar. Water boils at 75 degrees F at this pressure. How could an organism living on the surface survive? Another...
  38. J

    Equivalent Conditions for Nonsingularity (Matrices)

    True or False. If true explain or prove answer, and if false give an example to show the statement is not always true. 1. If A is a 4x4 matrix and a1+a2=a3+2a4, then A must be singular. 2. If A is row equivalent to both B and C, then A is row equivalent to B+C. My Work: 1. I say it's...
  39. mesa

    Solve a differential equation for the initial conditions,

    Homework Statement Solve y' = (y^3)(t^2) for the initial condition y(0)=0 and state in which interval in 't' this solution exists. The Attempt at a Solution First I divided both sides by y^3 and then subtracted t^2 from each as well, -t^2 + y'(y^-3) = 0 then solved, (-t^3)/3 +...
  40. A

    Expressing A for Orthogonal Vectors: Conditions on a,b & c

    Let a,b,c be three 3x1 vectors. Let A be a 3x3 upper triangular matrix which ensures that the 3x1 vectors d,e and f obtained using [d e f]=A[a b c] are orthogonal. a)Express the elements of A in terms of vectors a,b and c. b)what is the condition on a,b and c which allows us to find an...
  41. K

    Solving a PDE with Non-homogenous Boundary Conditions

    Homework Statement If utt - uxx= 1-x for 0<x<1, t>0 u(x,0) = x2(1-x) for 0≤x≤1 ut(x,)=0 for 0≤x≤1 ux(x,)=0 u(1,t)=0 find u(1/4,2) Homework Equations The Attempt at a Solution I was thinking to make a judicious change of variables that not only converts the PDE to a homogenous PDE, but also...
  42. M

    Uniqueness of the solution with certain boundary conditions

    Hey! Speaking electrodynamics, I can't seem to get mathematically or even physically convinced that the solution with Dirichlet or Neumann boundary conditions is UNIQUE. Can someone explain it? Thanks.
  43. B

    Uniform distribution of charge on a plane and on a bar: conditions to

    On the plane z=0 there is a superficial charge distribution such that \sigma is constant. Near to the plane, there is a bar, charged uniform with total charge q. At the extremities the bar has two constraints, so it can't turn. If I want to find the constraints force and the force momentum...
  44. S

    Initial conditions in quantum mechanics

    Hello, I'm doing an undergraduate introductory course in quantum mechanics, and I'm having a hard time understanding the interpretation. I've recently learned about the famous "Particle in a box" problem, in which we solve the time-independent schrodinger wave equation to get the energy...
  45. DrChinese

    Three-Particle Quantum Nonlocality under Strict Locality Conditions

    Just posted to the archives is a great new paper from a top experimental team. These are some of the same individuals that performed the now standard citation regarding a Bell test under strict locality conditions. They have now extended their concept to GHZ states, and also close the...
  46. J

    Doubts on the boundary conditions of PDE

    Hi all, Say I am solving a PDE as \frac{\partial y^2}{\partial^2 x}+\frac{\partial y}{\partial x}=f, with the boundary condition y(\pm L)=A. I can understand for the second order differential term, there two boundary conditions are well suited. But what about the first order differential term...
  47. DaTario

    Infinite Well and Boundary Conditions

    Hi All, I would like to know why in the infinite well problem, after having solved the time independent SE, we are not supposed to equal to zero the x derivative of the spatial part of the wave function at -L and L (2L being the total width). We only have to make it zero at the boundary...
  48. E

    Equivalent conditions for a measurability restriction?

    Let Z:= A^T, where T is a countable set and A is a finite set. Under the product topology, Z is a compact metrizable space. (As a special case, notice that Z could be the Cantor set). Given a closed set X \subseteq Z, I'm interested in answering the question, "Does there exist some partition...
  49. M

    Under what conditions is a current generated w/ a magnet and solenoid?

    Homework Statement When putting a magnet into a solenoid, under what conditions is a current generated? The Attempt at a Solution In order for a current to be generated, the magnet needed to be moving as it passed through the solenoid. This is due to Faraday’s law, which states that an...
  50. M

    Conditions for Surjective and Injective linear maps

    Hello, I'm not sure if this should go under the HW/CW section, since it's not really a homework question, just a curiosity about certain kinds of functions. My specific question is this: If M: U→V is injective and dim(U)=dim(V), does that imply that M is surjective (and therefore...
Back
Top